Difference between revisions of "2008 AMC 10A Problems/Problem 1"

(New page: ==Problem== A bakery owner turns on this doughnut machine at <math>8:30\ {\small\text{AM}}</math>. At <math>11:10\ {\small\text{AM}}</math> the machine has completed one third of the day's...)
 
m (redir)
 
Line 1: Line 1:
==Problem==
+
#redirect [[2008 AMC 12A Problems/Problem 1]]
A bakery owner turns on this doughnut machine at <math>8:30\ {\small\text{AM}}</math>. At <math>11:10\ {\small\text{AM}}</math> the machine has completed one third of the day's job. At what time will the doughnut machine complete the job?
 
 
 
<math>\mathrm{(A)}\ 1:50\ {\small\text{PM}}\qquad\mathrm{(B)}\ 3:00\ {\small\text{PM}}\qquad\mathrm{(C)}\ 3:30\ {\small\text{PM}}\qquad\mathrm{(D)}\ 4:30\ {\small\text{PM}}\qquad\mathrm{(E)}\ 5:50\ {\small\text{PM}}</math>
 
 
 
==Solution==
 
{{solution}}
 
 
 
==See also==
 
{{AMC10 box|year=2008|ab=A|before=First Question|num-a=2}}
 

Latest revision as of 23:30, 25 April 2008